Đến nội dung

viet nam in my heart

viet nam in my heart

Đăng ký: 22-02-2015
Offline Đăng nhập: Riêng tư
****-

#711775 Đề thi THPT QG 2018

Gửi bởi viet nam in my heart trong 29-06-2018 - 22:07

Em nhận xét dưới góc độ một người đã thi thật trong 90 phút. (Mã đề 102 bên trên)

  • Đề thi vẫn còn một số lỗi (Tính giới hạn của một biểu thức nhưng không biết n là gì và tiến tới đâu)
  • Có câu hỏi bắt học sinh phải ngộ nhận kiến thức. Cụ thể câu 27 với cách biểu diễn biểu thức $aln3+bln5+cln7$ và $a,b,c$ là số hữu tỷ thì phải biết được cách biểu diễn đó có duy nhất hay không. Sẽ chẳng có vấn đề gì nếu đề bài chỉ hỏi tính tích phân đó (Có lẽ người ra đề biết một số học sinh sẽ dùng mánh khoé bấm trực tiếp và thử đáp án nên mới đổi cách hỏi). Ở một mã đề khác ta lại phải dùng kết quả $e$ là số siêu việt
  • Đề thi không phù hợp với khoảng thời gian 90 phút
  • Chỉ sau 20 câu nhưng đề thi đã có những câu khó mất thời gian Ví dụ như câu 26 học sinh đã phải dùng kiến thức về Bất Đẳng thức chẳng kém gì câu 41 hay 37 phần  vận dụng cao
  • Những câu hỏi không phù hợp với kỳ thi trắc nghiệm. Nói là không phù hợp với kỳ thi trắc nghiệm vì nó đòi hỏi thời gian làm khá lâu vừa biến đổi tay vừa bấm máy tính. Vậy thì làm sao học sinh có đủ thời gian làm bài mà vừa làm vừa phải lo xem mình có biến đổi sai gì không. Chưa nói đến không có thời gian để kiểm tra xem mình có tô nhầm hay làm nhầm những câu hỏi dễ phần bên trên hay không
  • Điển hình câu số phức số 45 với ý tưởng giống hệt với câu 34 trong đề minh hoạ lần 2 của Bộ năm 2017 nhưng cho dưới dạng vừa xấu vừa phức tạp về biến đổi tay. Kể cả biết ý tưởng thì cũng đã phải mất 10 phút vừa làm vừa kiểm tra biến đổi vừa tính. Câu 47 về đồ thị thì quá phức tạp vừa làm lại phải kết hợp đoán và xem đáp án
  • Câu số 40 hỏi về phương trình vi phân, mặc dù đã xuất hiện ở nhiều đề thi thử của các trường hay đề minh hoạ của bộ nhưng có lẽ cũng không nên đưa vào đề thi thật 

Tổng kết:

  • Theo em thì đề thi còn lỗi, quá dài, phân bố số lượng các phần cũng không hợp lý. Số câu hỏi phần vận dụng và vận dụng cao nhiều dẫn đến việc có những bài về cùng một chủ đề. Thực sự thì bộ cần phải cân đối được thời gian và cho học sinh thời gian để kiểm tra lại bài chứ mặc dù rất cần thận nhưng em vẫn làm sai những câu hỏi ở mức độ dễ
  • Bên cạnh đó đề thi đã khắc phục được một số những câu hỏi mà chỉ cần bấm máy tính nhưng một số khác thì vẫn còn có thể dùng mẹo. 



#696883 Tuần 4 tháng 11/2017: đường thẳng $AM$ luôn đi qua một điểm cố định...

Gửi bởi viet nam in my heart trong 20-11-2017 - 14:27

Bài 1. Cho tam giác $ABC$ nội tiếp trong đường tròn $(O)$ cố định với $B, C$ cố định và $A$ di chuyển trên $(O)$. $E,F$ lần lượt đối xứng $B,C$ qua $CA,AB$. $M$ là trung điểm $EF$. Chứng minh rằng đường thẳng $AM$ luôn đi qua một điểm cố định khi $A$ di chuyển.

 

attachicon.gifScreen Shot 2017-11-19 at 10.06.47 PM.png

geogebra-export.png

Kẻ đường cao $BX,CY$. Gọi $H$ là trực tâm tam giác $ABC$.

Do $\widehat{EAB}=\widehat{FAC}$ nên $2$ đường đẳng giác trong góc $\widehat{EAF}$ cũng đẳng giác góc $\widehat{BAC}$.

Ta sẽ chứng minh đường đối trung góc $A$ của tam giác $EAF$ đi qua giao điểm $2$ tiếp tuyến tại $B,C$ của $(HBC)$ cố định

Nghịch đảo cực $A$ phương tích $\overline{AX}.\overline{AC}$ ta thu được bài toán sau: Cho tam giác $ABC$ nội tiếp $(O)$ và điểm $I$ trên phân giác góc $A$. Giả sử $(IAB),(IAC)$ lần lượt cắt lại $AC,AB$ tại điểm thứ hai lần lượt là $X,Y$. Chứng minh rằng đường thẳng qua $I$ và trung điểm của $XY$ đi qua giao điểm $2$ tiếp tuyến tại $B$ và $C$ của $(O)$

2.png ]

Giả sử $(IAB),(IAC)$ cắt lại $BC$ tại $R,S$. Gọi $M,N,K$ lần lượt là trung điểm của $BC,XY,RS$

Gọi $O'$ là tâm của $(AXY)$. $U,V$ là điểm chính giữa cung $BC$ chứa $A$ và không chứa $A$ của $(O)$

Giả sử $NI$ cắt trung trực của $BC$ tại $T$. Ta chứng minh $T$ là giao điểm $2$ tiếp tuyến tại $B,C$ của $(O)$

Ta có: $\widehat{IRS}=\dfrac{\widehat{BAC}}{2}=\widehat{ISR} \Rightarrow IR=IS $

$O'B^2-O'C^2=BS.BC-CR.CB=BC(BS-CR)=(BK+CK)(BK-CK)=KB^2-KC^2$

Suy ra $O'K \perp BC$ hay $O'I \perp BC$.

Ta có: $O'O \perp AV, AV \perp AU$ nên $O'O \parallel UI$. Và $O'I \parallel OU( \perp BC)$ nên $O'OUI$ là hình bình hành

Dễ có: $BY=CX$ nên $V \in (O') \Rightarrow \triangle VYX \sim \triangle VBC$

Áp dụng định lý $Thales$ ta có: $\dfrac{VO}{VT}=\dfrac{O'I}{VT}=\dfrac{NO'}{NV}=\dfrac{MO}{MV}$

Suy ra $\dfrac{OV}{OT}=\dfrac{OM}{OV}$ hay $OM.OT=OV^2$. Suy ra $T$ là giao điểm $2$ tiếp tuyến tại $B,C$ của $(O)$

Trở lại bài toán: $AM$ cắt trung trực $BC$ tại $X$. Gọi $Y$ là giao điểm $2$ tiếp tuyến tại $B,C$ của $(BHC)$

Do $AX,AY$ đẳng giác trong góc $A$ nên $\overline{OX}.\overline{OY}=R^2=const$ và $Y$ cố định nên $X$ cố định

Tóm lại $AM$ đi qua điểm $Y$ cố định




#692510 Tồn tại một số nguyên tố cùng nhau với $n-1$ số còn lại.

Gửi bởi viet nam in my heart trong 06-09-2017 - 23:04

Chứng minh rằng trong $n$ ($n\geq 2$) số nguyên dương liên tiếp, tồn tại một số nguyên tố cùng nhau với $n-1$ số còn lại.

Vấn đề này được đề cập trong phần đầu ở cuốn sách nổi tiếng của Sierpinski(https://diendantoanh...bản-tiếng-việt/  Trang 5) và mình cũng từng đăng lên diễn đàn. Số lớn nhất thỏa mãn tính chất trên là $16$ còn lại nếu $n \geq 17$ thì đều tìm được phản ví dụ.

Mới đây mình tìm được các links sau mọi người tham khảo:https://projectpen.f...a9-a37-o511.pdf

https://artofproblem...ity/c146h150894




#683764 Đề thi vào 10 chuyên tỉnh Vĩnh Phúc 2017-2018

Gửi bởi viet nam in my heart trong 09-06-2017 - 11:03

c) Áp dụng định lí $Ptolemy$ cho tứ giác $ABEC$ có: $AB.CE+AC.BD=AE.BC \iff AB(CE+1)=AE\\ \iff CD+1=\dfrac{AE}{AB}=2\dfrac{CF}{CD}$

Do đó: $CF=\dfrac{CD(CD+1)}{2}$

Mà: $EF$ là phân giác $\widehat{BEC} \implies \dfrac{CF}{FB}=\dfrac{CE}{EB}=CE=CD$

Thay vào hệ thức: $CD+1=2FB \implies FB=\dfrac{CD+1}{2}$

Giải phương trình: $\dfrac{CD(CD+1)}{2}+\dfrac{CD+1}{2}=1 \iff CD=\sqrt{2}-1$

a.png

Phần $c$ có thể giải như sau độc lập với hai phần $a,b$ và không cần đến định lý $Ptolemy$

Gọi $M,N$ là trung điểm $BC,DC$.$DE$ cắt $BC$ tại $H$. Đặt $CD=x(x>0)$

Do tính đối xứng nên $\widehat{BEC}=\widehat{BDC}=\widehat{BCD}$ nên $CD$ là tiếp tuyến của $(ABC)$

Do tam giác $ABC$ cân tại $A$ và $AD \parallel BC$ nên $AD$ là tiếp tuyến của $(ABC)$. Từ đó $DC=DA$

Ta có: $ADHM$ là hình chữ nhật nên $DC=DA=HM=CM-CH=\dfrac{1}{2}-CH$. Từ đó $CH=\dfrac{1}{2}-x$

Tam giác $BDC$ cân tại $B$ và $N$ là trung điểm $CD$ nên $BN \perp CD$ và khi đó: $BDNH$ nội tiếp.

Do đó $CN.CD=CH.CB\Leftrightarrow \dfrac{x^2}{2}=\dfrac{1}{2}-x$. Từ đó tìm được $x= \sqrt{2}-1$

(Đẳng thức trên có thể một số nơi không cho phép dùng với bậc THCS. Có thể cần phải chứng minh. Ý này phù hợp vào đề thi $MTCT$ hơn là vào đề thi chuyên)

Sao bạn lại không làm đc câu cuối nhỉ :D

mà mk nói thêm nhé: phương trình có vô số nghiệm nguyên dương cx chưa chắc là nó >2017 đâu, cần làm thêm 1 chút nữa để c/m nó >2017

Bạn có thể post lời giải câu cuối được không :) 




#679915 Đề thi Olympic chuyên KHTN 2017

Gửi bởi viet nam in my heart trong 07-05-2017 - 22:48

 

Ngày 2

 

Câu 5. Tìm tất cả các bộ số nguyên không âm $(m,n,k)$ thỏa mãn

\[k^2-k+4=5^m(2+10^n)\]

 

Câu 6. Cho tam giác $ABC$ nhọn, không cân, nội tiếp trong đường tròn $(O)$. $I$ là tâm nội tiếp của tam giác $ABC$. $AI$ cắt $BC$ tại $D$ và cắt $(O)$ tại $K$ khác $A$. $P$ là một điểm nằm trên đường tròn ngoại tiếp tam giác $BIC$ và nằm trong tam giác $ABC$. $PK$ cắt $BC$ tại $L$. $AL$ cắt $(O)$ tại $F$ khác $A$. Giả sử $KF$ cắt $BC$ tại $T$. $Q$ đối xứng $P$ qua $K$. $AQ$ cắt $(O)$ tại $R$ khác $A$.

 
a) Chứng minh rằng $PT$ song song với $KR$.
 
b) Gọi giao điểm của $AP$ và $(O)$ là $E$ khác $A$. Chứng minh rằng hai tam giác $KEP$ và $KET$ có diện tích bằng nhau. 

 

Câu 7. Giả sử $A=(a_1,a_2,...,a_n)$ gồm các số thuộc tập $M= \lbrace 1,2,...,m \rbrace $ sao cho $a_1+a_2+a_3+...+a_n=2S$ với $S$ là số nguyên chia hết cho mọi phân tử của $M$. Chứng minh rằng người ta có thể chọn từ $A$ một số số có tổng bằng $S$

 

 

Câu 6:

a.png

Nhận thấy việc chứng minh $PT \parallel KR$ và $QT \parallel KE$ là như nhau (Do vai trò của $P,Q$ là như nhau). Hơn nữa nếu có $QT \parallel KE$ kết hợp với $K$ là trung điểm của $PQ$ thì $KE$ là đường trung bình của tam giác $PQT$ nên suy ra $KE$ đi qua trung điểm $QT$ thì từ đó cũng suy ra phần $b$ của bài toán

Ta chứng minh $QT \parallel KE$ là đủ

Ta có: $\overline{LP}.\overline{LQ}=\overline{LB}.\overline{LC}=\overline{LA}.\overline{LF}$. Từ đó $APFQ$ nội tiếp

Do $BC$ và $(O)$ là ảnh của nhau qua phép nghịch đảo cực $K$ phương tích $KI^2=KP^2=KQ^2$ nên $KQ^2=\overline{KF}.\overline{KT}$

Từ đó $QK$ tiếp xúc $(QTF)$

Ta có: $\widehat{QTK}=\widehat{FQP}=\widehat{FAP}=\widehat{FKE}$. Suy ra $QT \parallel KE$ (đpcm)

(Nếu không gọi ra 2 điểm $T$ và $Q$ thì có thể yêu cầu chứng minh $KE$ là đường đối trung của tam giác $PKF$)

Về bài toán $7$, mình nghĩ nó là biến thể của bài toán sau:

(Tournament of towns) Có một số tấm thẻ mà trên đó điền các số nguyên không vượt quá $n$ sao cho tổng các số ghi trên các tấm thẻ này là $k\cdot n!$. Chứng minh rằng ta có thể chia các tấm thẻ này thành $k$ đống, mỗi đống có tổng các số ghi trên các tấm thẻ đúng bằng $n!$.

Có lẽ bạn nên đăng cả nguồn lẫn lời giải để dễ theo dõi.

Câu 7: Đây là một bài trong đề thi học sinh giỏi tỉnh mình. Chỉ cần $a_1+a_2+a_3+...+a_n \geq 2S$ là đủ chứ không nhất thiết cần đẳng thức.

Sau đây là lời giải của đáp án.

Nếu mỗi số từ $1$ đến $m$ chỉ xuất hiện tối đa $m-2$ lần thì $a_1+a_2+a_3+...+a_n \leq (m-2)(1+2+\ldots+m)<2S$ (mâu thuẫn)

Từ đó trong số các số trong $M$ có một số được xuất hiện ít nhất $m-1$ lần. Giả sử đó là số $a$

Bỏ $m-1$ số $a$ vào tập $C$ và còn lại $n-m+1$ số bỏ vào tập $B$

Nếu $n-m+1<a$ thì $n-m+1<m \Rightarrow n<2m-1$. Suy ra $a_1+a_2+a_3+...+a_n < m(2m-1)<2S$ (Mâu thuẫn)

Suy ra $n-m+1 \geq a$

Trong tập $B$ lấy ra $a$ số bất kỳ

Bổ đề: Cho số nguyên dương $a$ thì trong $a$ số nguyên bất kỳ ta luôn chọn được $1$ số hoặc một số số có tổng chia hết cho $a$ (chứng minh đơn giản bằng Đirichlet)

Áp dụng bổ đề thì trong $a$ số kia chọn được $1$ số hoặc một số số có tổng chia hết cho $a$. Bỏ những số này ra khỏi $B$ và đặt chúng vào tập $T$

Ta tiếp tục làm như trên đến khi tổng các số trong $T$ lớn hơn $S-ma$ lần đầu tiên thì dừng lại

(Do nếu tổng các số trong $T$ nhỏ hơn hoặc bằng $S-ma$ thì tổng các số trong $B$ sẽ lớn hơn hoặc bằng $2S-(m-1)a-(S-ma)=S+a>ma$. Tức là trong $B$ vẫn còn nhiều hơn $a$ số để thực hiện tiếp)

Do đó ta đã xây dựng được tập $T$ có: $\sum T \vdots a$ và $\sum T >S-ma$ ( Ký hiệu $\sum T$ là tổng các phần tử của tập $T$)

Mặt khác $S \vdots a$ nên $\sum T \geq S - (m-1)a$

Ta chứng minh $\sum T \leq S$.

Giả sử  $\sum T > S$ thì $\sum T \geq S+a$. Do một lần thực hiện bước chuyển số vào $T$ thì ta chỉ chuyển được tối đa $a$ số vào $T$ nên sau mỗi bước chuyển thì $T$ tăng tối đa là $ma$ (Khi chuyển $a$ số $m$ vào $T$). Do đó nếu không chuyển lần chuyển cuối cùng thì $\sum T \geq S+a-ma=S-(m-1)a>S-ma$ nên ta có điều mâu thuẫn ( Do ta đã giả sử khi dừng lại khi $\sum T >S-ma$ lần đầu tiên)

Do đó ta tìm được tập $T$ mà $S-(m-1)a\leq\sum T\leq S$ và $\sum T \vdots a$

Mặt khác ta có tập $C$ chứa $m-1$ số $a$ nên ta sẽ bổ sung một số số $a$ vào $T$ để $\sum T =S$

Vậy ta có thể chọn từ $A$ một số số có tổng bằng $S$ 

(Lưu ý là một số bất đẳng thức trong bài trên chỉ đúng từ $m \geq 4$ nên ta cần xét trường hợp $m \leq 3$. Những trường hợp này khá đơn giản)




#675378 ĐỀ VIỆT NAM TST 2017

Gửi bởi viet nam in my heart trong 26-03-2017 - 16:36

Bài 2. Với mỗi số nguyên dương $n$, đặt $x_n = C_{2n}^n$.

a) Chứng minh rằng nếu $\dfrac{2017^k}{2} < n < 2017^k$ với $k$ là số nguyên dương nào đó thì $x_n$ là bội của $2017$.
b) Tìm tất cả số nguyên dương $h > 1$ để tồn tại các số nguyên dương $N,T$ sao cho với mọi $n>N$ thì $x_n$ là dãy số tuần hoàn theo modulo $h$ với chu kỳ $T$.

 

a)Ta áp dụng định lý Lucas: 

Từ điều kiện đề bài đặt $n=y_{k-1}2017^{k-1}+...+y_{1}2017+y_0$(biểu diễn trong hệ cơ số $2017$)

Nếu $y_{k-1},...,y_1,y_0 \leq 1008$ thì ta có $n\leq 1008\left(2017^{k-1}+ \ldots + 1\right) < \dfrac{2017^k}{2}$ (loại)

Từ đó có một vị trí $y_{i}$ nào đó sao cho $y_i \geq 1009$. (gọi $i$ là vị trí đầu tiên như vậy trong biểu diễn cơ số $2017$ của $n$ tính từ bên phải)

Khi đó xét $2n$ thì vị trí đó có giá trị là $2n-2017$. Mặt khác $2n-2017 < n$ nên $C_{2n-2017}^n=0$ nên rõ ràng theo định lý $Lucas$ thì ta có điều phải chứng minh

b) Gọi $p$ là một ước nguyên tố của $h$

Nếu $x_n$ tuần hoàn theo modulo $h$ với $n$ đủ lớn thì nó cũng tuần hoàn theo modulo $p$ với $n$ đủ lớn

Mặt khác từ phần a thì rõ ràng ta chỉ được một khoảng các số $n$ liên tiếp sao cho $x_n$ chia hết cho $p$ với mọi $n$ thuộc khoảng này. Mà ta chọn được khoảng này đủ lớn về cả vị trí lẫn độ dài nên ta suy ra với mọi $n$ đủ lớn $\geq N_0$ thì $x_n$ phải chia hết cho $p$

Nếu $p>2$

Ta chọn số $n=111..1$($k$ số $1$) trong hệ cơ số $p$ thì theo định lý $Lucas$ thì $x_n$ chia $p$ dư $2^k$ suy ra $x_n$ không chia hết cho $p$ suy ra vô lý

Từ đó $p=2$

Do đó $h=2^t$ với $t$ là một số nguyên dương nào đó 

Ta chứng minh cũng tồn tại một khoảng đủ lớn về vị trí lẫn độ dài để $x_n$ chia hết cho $2^t$ với mọi $n$ thuộc khoảng này

Ta có: $x_n=C_{2n}^n=\dfrac{\left(2n\right)!}{\left(n!\right)^2}$

Do đó áp dụng công thức tính số mũ đúng của $2$ thì ta được $v_{2}(x_n)=2n-s_{2}(2n)-2n+2s_{2}(n)=2s_{2}(n)-s_{2}(2n)$

với $s_{2}(n)$ là tổng các chữ số trong biểu diễn cơ số $2$ của $n$

Ta cũng có thể thu gọn được $2s_{2}(n)-s_{2}(2n)=z$ với $z$ là số chữ số $1$ trong biểu diễn cơ số $2$ của $n$ 

  • Xét trong hệ cơ số $2$ 

Do đó ta xét khoảng các số liên tiếp từ $11..10..0$ ($u$ số $1$ đến $v$ số $0$) đến $11...1$ gồm có $u+v$ số $1$ thì khi chọn $u,v$ đủ lớn ta đều thu được $x_n$ chia hết cho $2^t$ với mọi $n$ thuộc khoảng này.

Chọn $u,v$ đủ lớn để độ dài khoảng này lớn hơn $2T$ và $11..10..0>N$ ($u$ số $1$ và $v$ số $0$)

Từ đó với mọi $n$ lớn hơn $N$ thì $x_{n}$ phải chia hết cho $2^t$

Mặt khác ta chọn $n=100...0$ với số số $0$ đủ lớn để $n$ đủ lớn thì $v_{2}(x_n)=2s_{2}(n)-s_{2}(2n)=1$ nên rõ ràng $t$ chỉ có thể bằng $1$

Khi đó ta dễ chứng minh mọi số hạng $x_n$ đều chia hết cho $2$

Vậy $h=2$




#671266 VMF's Marathon Hình học Olympic

Gửi bởi viet nam in my heart trong 12-02-2017 - 13:04

Theo đề nghị của anh Khánh, em đề xuất bài toán tiếp theo, có lẽ là của anh Phạm Hy Hiếu, HCB IMO năm 2009.

 

Bài toán 168. Cho tam giác $ABC$ nội tiếp $(O)$ có $BC>CA>AB$ và $I$ là tâm nội tiếp. $AI$ cắt lại $(O)$ tại $K$. $M$ là trung điểm $BC$. Gọi $N$ đối xứng với $I$ qua $M$. $KN$ cắt lại $(O)$ tại $L$. Chứng minh rằng $LB=LC+LA$.

Mãi mới có thời gian rảnh để làm. Bài này vẽ hình phụ khá đơn giản nhưng rất đẹp. 

abc.png

Trên $LB$ lấy $S$ sao cho $LS=LA$.

Ta có: $\widehat{ASB}=180^o-\widehat{ASL}=180^o-\dfrac{180^o-\widehat{ALS}}{2}=90^o+\dfrac{\widehat{ALS}}{2}=90^o+\dfrac{\widehat{ACB}}{2}=\widehat{AIB}$

Từ đó tứ giác $AISB$ là tứ giác nội tiếp. Lấy $L'$ đối xứng với $L$ qua $M$. Theo tính đối xứng thì $LC=L'B$

Ta có: $\widehat{ISL}=\widehat{BAI}=\widehat{BLK}$ nên $IS \parallel KL$. Mà do tính đối xứng thì $IL' \parallel KL$ nên $I,L',S$ thẳng hàng

Do cần chứng  minh :$LB=LC+LA$ mà $LS=LA$ và $LC=L'B$ nên chỉ cần chứng minh $BS=BL'$

Hay cần chứng minh tam giác $BSL'$ cân tại $B$. Tương đương: $\widehat{L'BS}=180^o-2\widehat{BSL'}=180^o-2\widehat{BAI}=180^o-\widehat{BAC}$

Mà do $BL' \parallel LC$ nên $\widehat{L'BS}=180^o-\widehat{BLC}=180^o-\widehat{BAC}$

Từ đó tam giác $BSL'$ cân tại $B$

Kết hợp những điều trên ta có điều phải chứng minh $\blacksquare$




#670242 VMF's Marathon Hình học Olympic

Gửi bởi viet nam in my heart trong 28-01-2017 - 17:43

Cám ơn Dương lời giải của em cũng tương tự lời giải của Luis đưa ra ở đây nên thầy không dịch lại lời giải trong link nữa. Xin đề nghị bài tập tiếp

 

Bài toán 146. Cho tam giác $ABC$ với trực tâm $H$. $D$ là điểm nằm cũng phía $A$ với $BC$. $E,F$ thuộc cạnh $CA,AB$ sao cho $\angle HEC=\angle DCB$ và $\angle HFB=\angle DBC$. $M,N$ là đối xứng của $E,F$ qua $HC,HB$. Đường tròn $(HCM)$ và $(HBN)$ cắt nhau tại $K$ khác $H$. $L$ là tâm ngoại tiếp của tam giác $HEF$. Chứng minh rằng $HL\parallel KD$.

 

attachicon.gifFigure4286.png

Bài này hay quá. Bài này chắc ý tưởng là từ $APMO$ $2010$ và $Tuymada$ $2009$

mmj.png

Ta có: $\widehat{BKC}=\widehat{BKH}+\widehat{CKH}=\widehat{BNH}+\widehat{CMH}=\widehat{BFH}+\widehat{CEH}=\widehat{DBC}+\widehat{DCB}=180^{o}-\widehat{BDC}$

Từ đó $BDCK$ nội tiếp. Lấy $G$ sao cho $BGDC$ là hình thang cân. từ đó $B,G,D,C,K$ cùng nằm trên một đường tròn

Đặt $\widehat{BFH}=\alpha$ và $\widehat{CEH}=\beta$. Giả sử $GB,GC$ cắt lại $(HBN)$ và $(HCM)$ lần lượt tại $I,J$

Ta biến đổi các góc sau: $\widehat{BHN}=\widehat{BHF}=180^{o}-\widehat{BFH}-\widehat{FBH}=90^{o}+\widehat{A}-\alpha$

Tương tự: $\widehat{CHM}=90^{o}+\widehat{A}-\beta$. Từ đó $\widehat{MHN}=\widehat{BHN}+\widehat{CHM}-\widehat{BHC}=3\widehat{A}-\alpha-\beta$

Lại có: $\widehat{IHN}=180^{o}-\widehat{GBN}=180^{o}-\widehat{GBC}-\widehat{NBH}+\widehat{HBC}=180^o+\widehat{A}-\widehat{C}-\beta$

Tương tự :$\widehat{JHM}=180^o+\widehat{A}-\widehat{B}-\alpha$

Do đó $\widehat{IHJ}=\widehat{JHM}+\widehat{IHN}-\widehat{MHN}=180^o$. Suy ra $I,H,J$ thẳng hàng

Khi đó :$\widehat{GIJ}=\widehat{HNB}=\widehat{HFB}=\widehat{GCB}$. Suy ra $BIJC$ nội tiếp 

Xét trục đẳng phương của $(BIJC),(HBN),(HCM)$ suy ra $G,H,K$ thẳng hàng

Gọi $X,Y,U,V$ lần lượt là tâm của $(BHN),(CHM),(BHF),(CHE)$

Ta sẽ chứng minh $HL \parallel KD$ bằng biến đổi góc.

Sau khi biến đổi ta thấy chỉ cần chứng minh $\widehat{HEF}=\widehat{HCK}$ và $\widehat{HFE}=\widehat{HBK}$ (Đoạn này em biến đổi góc hơi loằng ngoằng nên xin phép không ghi rõ ra)

Ta chuyển về chứng minh tam giác $KIJ$ và tam giác $HFE$ đồng dạng

 Mặt khác do $H,I,J$ thẳng hàng nên ta dễ dàng chứng minh được tam giác $KIJ$ và tam giác $KXY$ đồng dạng với nhau 

Do đó chỉ cần chứng minh tam giác $KXY$ và tam giác $HFE$ đồng dạng

Mà : $\widehat{XKY}=\widehat{IKJ}=\widehat{IBH}+\widehat{JCH}=\alpha+\beta-\widehat{A}$. Ta cũng có $\widehat{FHE}=\alpha+\beta-\widehat{A}$

Từ đó chỉ cần chứng minh $\dfrac{KX}{HF}=\dfrac{KY}{HE}$

Do tính đối xứng nên $KX=HU,KY=HV$

Từ đó $\dfrac{KX}{HF}=\dfrac{HU}{HF}=\dfrac{2}{sin \widehat{ABH}}=\dfrac{2}{sin \widehat{ACH}}=\dfrac{HV}{HE}=\dfrac{KY}{HE}$

Kết hợp những điều trên thì ta hoàn tất chứng minh 

Bài toán 154: Cho hai đường tròn $(O;R)$ và $(I;r)$ sao cho $OI^2=R^2-2Rr$. Khi đó tồn tại tam giác $ABC$ thay đổi nội tiếp $(O)$ và ngoại tiếp $(I)$. Chứng minh tâm đẳng phương của $3$ đường tròn bàng tiếp của tam giác $ABC$ thuộc một đường tròn cố định




#669051 VMF's Marathon Hình học Olympic

Gửi bởi viet nam in my heart trong 20-01-2017 - 18:17

Bài toán 136. Cho tam giác $ABC$ nội tiếp đường tròn $(O)$ đường tròn bàng tiếp góc $A$ là $(J)$ tiếp xúc $BC$ tại $D$. Đường tròn qua $A,B$ tiếp xúc $(J)$ tại $M$. Đường tròn qua $A,C$ tiếp xúc $(J)$ tại $N$. $BM$ cắt $CN$ tại $P$. Chứng minh rằng $\angle PAB=\angle DAC$.

 

Lời giải bài toán 136

kl.png

Gọi $D,E,F$ là tiếp điểm của $(J)$ trên $BC,CA,AB$

Gọi $X,Y,Z$ là trung điểm $EF,DE,DF$

Ta chứng minh $BMXF$ và $CNXE$ nội tiếp 

Thật vậy nghịch đảo cực $A$ phương tích bất kỳ thì được cấu hình: tam giác $ABC$ với $(X)$ là đường tròn $A-Mixtilinear$ tiếp xúc với $AB,AC$ lại $F,E$. Tiếp tuyến thứ hai $BM$ đến $(K)$. Khi đó dễ thấy $B,M,X,F$ nội tiếp đường tròn

Từ đó nghịch đảo lại thì ở bài toán này $BMXF$ và $CNXE$ nội tiếp 

Xét phép nghịch đảo cực $D$ phương tích $JD^2$ thì ta thu được $AMZF$ và $ANYE$ nội tiếp

$AD$ cắt $EF$ tại $T$. $S$ đối xứng với $T$ qua $X$ thì do tam giác $AEF$ cân tại $A$ nên $AS,AT$ đẳng giác 

Do đó ta chứng minh $AS,BM,CN$ đồng quy. Xét cực và đối cực với $(J)$. Gọi $P,Q,R$ là cực của $AS,BM,CN$ với $(J)$.

Cần chứng minh $P,Q,R$ thẳng hàng

Khi đó theo định lý $Lahire$ thì $P,Q,R$ lần lượt nằm trên $EF,DF,DE$ nên ta sẽ áp dụng định lý $Menelaus$ để chứng minh

Thật vậy $(PSFE)=-1$ nên $\dfrac{PE}{PF}=\dfrac{SE}{SF}=\dfrac{TF}{TE}=\dfrac{DF^2}{DE^2}$ ( do $AD$ là đối trung của tam giác $DEF$)

Lại có: $Q$ là đối cực của $BM$ nên $QM$ tiếp xúc $(J)$. Từ đó :$\dfrac{QF}{QD}=\dfrac{MF^2}{MD^2}$

Giả sử $MZ$ cắt lại $(J)$ tại $L$. Ta có: $\widehat{AFZ}=\widehat{FLD}$ (góc tiếp tuyến) và $\widehat{FDL}=\widehat{FML}=\widehat{FAZ}$

Suy ra tam giác $FLD$ và tam giác $ZFA$ đồng dạng. Kết hợp $Z$ là trung điểm $DF$ nên ta thu được: $\dfrac{QF}{QD}=\dfrac{MF^2}{MD^2}=\dfrac{LD^2}{LF^2}=\dfrac{FA^2}{FZ^2}$

Tương tự: $\dfrac{RD}{RE}=\dfrac{EY^2}{EA^2}$. Từ đó $\dfrac{PE}{PF}.\dfrac{QF}{QD}.\dfrac{RD}{RE}=1$

Vậy ta có điều phải chứng minh

Ngoài ra ta hoàn toàn có thể chứng minh được $BMXF$ và $AMZF$ nội tiếp mà hoàn toàn không cần đến phép nghịch đảo nhưng lời giải sẽ dài hơn

Bài toán 137. Cho tam giác $ABC$ có $AC>BC>AB$. Đường tròn bàng tiếp góc $A$ và đường tròn nội tiếp tam giác $ABC$ tiếp xúc với đường tròn $Euler$ của nó tại $T,T_a$. Gọi $D,E$ là hình chiếu của $B,C$ lên $AC,AB$. Gọi $M,N$ là trung điểm $AC,AB$. Chứng minh $MN,DE,TT_a$ đồng quy 




#668817 VMF's Marathon Hình học Olympic

Gửi bởi viet nam in my heart trong 19-01-2017 - 01:05

Bài toán 134. Cho tam giác $ABC$ và $DEF$ là tam giác pedal của $P$ bất kỳ. $(DEF)$ cắt $BC$ tại $G$ khác $D$. Đường thẳng qua $P$ vuông góc với $EF$ cắt $DE,DF$ tại $M,N$. Đường tròn $(DMN)$ cắt $(DEF)$ tại $Q$ khác $D$. Lấy $T$ sao cho $TM\perp AC,TN\perp AB$. $AT$ cắt $BC$ tại $S$. Chứng minh bốn điểm $A,Q,G,S$ đồng viên.

k.png

Gọi $J$ liên hợp đẳng giác với $P$ thì $AJ \perp EF$ suy ra $AJ \parallel MN$

Biến đổi góc: $(MN,MT)=(MN,AC)+(AC,MT)=(AJ,AC)+\dfrac{\pi}{2}=(AB,AP)+\dfrac{\pi}{2}=(EF,EP)+(EP,EA)$

Suy ra $(MN,MT)=(EF,EA)$. Từ đó $\widehat{TMN}=\widehat{AEF}$

Suy ra tam giác $TMN$ và $AEF$ đồng dạng 

Kết hợp $Q$ là điểm $Miquel$ ứng với $L,M,N$ của tam giác $DEF$ nên  phép vị tự quay tâm $Q$ góc quay $\dfrac{\pi}{2}$ biến $M$ thành $E$, biến $N$ thành $F$, biến $T$ thành $A$

Từ đó ta có $3$ tam giác đôi một đồng dạng $ATQ,FNQ,EMQ$. Suy ra $\widehat{TAQ}=\widehat{NFQ}=\widehat{DGQ}$

Suy ra điều phải chứng minh

Em đề nghị bài toán $135$ như sau

Bài toán 135(Lê Bá Khánh Trình- Chuyên đề toán học số 10) 

Cho tam giác $ABC$ nội tiếp $(O)$ có trung tuyến $AD$, trực tâm $H$. Giả sử $P$ di động trên $AD$. Đường thẳng $BP,CP$ cắt lại $(O)$ tại $M,N$. Gọi $R,S$ lần lượt là điểm đối xứng với $M,N$ qua trung điểm các cạnh $AC,AB$. Chứng minh rằng tâm đường tròn ngoại tiếp tam giác $HRS$ luôn thuộc một đường cố định khi $P$ di động 




#667679 P, Q, H, S cùng thuộc 1 đường tròn

Gửi bởi viet nam in my heart trong 08-01-2017 - 21:52

Cho tứ giác ABCD nội tiếp trong đường tròn tâm O. Một đường tròn tâm I tiếp xúc với đường thẳng AB, CD lần lượt tại N, M. (I) cắt (O) tại 2 điểm H và S. AC, BD cắt MN lần lượt tại Q, P. Chứng minh: P, Q, H, S cùng thuộc 1 đường tròn và đường tròn này tiếp xúc với AC và BD.

Đây là một áp dụng của bổ đề Poncelet

Có thể xem thêm trong tài liệu sau của tác giả Trần Minh Ngọc: https://tranminhngoc...81-poncelet.pdf




#667676 Đề cử Thành viên nổi bật 2016

Gửi bởi viet nam in my heart trong 08-01-2017 - 21:39

1)Tên nick ứng viên: JUV, bangbang1412,Ego, Zaraki, baopbc

2)Thành tích nổi bật:Khởi xướng phong trào Marathon trên diễn đàn: Số học:(bangbang1412,Ego, Zaraki),

                                        Đa thức (bangbang1412,Ego), Hình học (baopbc)

                                         Tích cực giải các bài toán tổ hợp JUV

3)Ghi chú:không




#658859 Đề chọn đội tuyển học sinh giỏi quốc gia tỉnh Vĩnh Phúc (ngày 2) 2016-2017

Gửi bởi viet nam in my heart trong 23-10-2016 - 00:23

Bài 1. Xét $\lim |b_n|>0$. Khi đó áp dụng định nghĩa giới hạn dễ chứng minh tồn tại $q>0$ sao cho tồn tại vô hạn số $|b_n|>q$

Khi đó hiển nhiên $c_n$ sẽ tăng lên vô hạn.

Xét $\lim |b_n|=0$, khi đó $\forall \epsilon>0$, tồn tại $n_0$ sao cho $\forall n>n_0$ thì $|a_{n+1}-a_{n}|<\epsilon a_n$

Từ đó suy ra $(1+\epsilon)a_n>a_{n+1}$, từ đó sẽ tồn tại $n_1$ sao cho với $n\geqslant n_1$ thì $a_n\geqslant a_{n+1}$

Xét $n\geqslant n_1$ thì $b_n=\dfrac{a_n-a_{n+1}}{a_n}\geqslant \dfrac{a_n-a_{n+1}}{a_{n_1}}$

Từ đó ta có $|b_{n_1}|+|b_{n_1+1}|+...+|b_n|\geqslant 1-\dfrac{a_{n+1}}{a_{n_1}}$

Do $\lim a_{n} = 0$ nên ta có thể chọn $n=n_2-1>n_1$ để $1-\dfrac{a_{n+1}}{a_{n_1}}>q>0$

Xét $n\geqslant n_2$ thì $|b_{n_2}|+|b_{n_2+2}|+...+|b_n| \geqslant 1-\dfrac{a_{n+1}}{a_{n_2}}$

Khi đó lại chọn $n=n_3-1>n_2$ để $1-\dfrac{a_{n+1}}{a_{n_2}}>q>0$

Cứ tiếp tục quá trình trên thì $c_{n_k-1}\geqslant (k-1)q$ nên $c_{n}$ tăng lên vô hạn.

Em chưa đọc đoạn sau nhưng muốn như trên cần chứng minh thêm $|b_n|$ có giới hạn. Khi nó có giới hạn vô cực có lẽ viết là $>0$ cũng không hợp lý lắm




#657694 Đề chọn đội tuyển học sinh giỏi quốc gia tỉnh Quảng Ninh ngày 2 2016-2017

Gửi bởi viet nam in my heart trong 12-10-2016 - 22:30

Bài 2: Khá đơn giản

Dễ thấy $degP \geq 1$. Cho $n$ đủ lớn thì $n+d(P(n))>n$ nên $P(n+d(n))=n+d(P(n)) \to +\infty$

Do đó hệ số cao nhất của $P$ là một số nguyên dương

TH1: $P(n)$ lẻ với mọi $n$.

Khi đó chọn $n$ lẻ thì $n+d(P(n))$ là số chẵn nên $P(n+d(n))$ là số chẵn suy ra mâu thuẫn với trên (loại)

TH2: Tồn tại $t$ để $P(t)$ là số chẵn 

Do $P(t+2)-P(t)$ chia hết cho $2$ nên cứ như vậy tồn tại vô hạn $t$ nguyên dương để $P(t)$ chẵn

Khi đó $d(P(t))=2$ nên thay vào thì $P(t+d(t))=t+2$ (2)với vô hạn $t$

-Nếu $degP >2$ thì ta có nhận xét quan trọng sau $\dfrac{P(n)}{n} \to +\infty$ khi $n \to +\infty$ (hệ số cao nhất của $P$ dương)

Do đó hoàn toàn tồn tại $n_0$ để $P(n)>2n$ với mọi $n> n_0$ 

Do (2) đúng với vô hạn $t$ nguyên dương nên rõ ràng ta chọn được $t>n_0$

Khi đó $t+2=P(t+d(t))>2(t+d(t))>2(t+2)$ với vô hạn $t$ suy ra vô lý

Do đó $degP=1$. Cũng tương tự như trên dễ dàng ra được $P(x)=x-k$ với $k$ nguyên không âm và $\forall x \in \mathbb{R}$

Thay vào giả thiết thì $n+d(n)-k=n+d(n-k)$ với mọi $n$ nguyên dương, $n>1$

Hay $d(n)-k=d(n-k)$  với mọi $n$ nguyên dương, $n>1$

Chọn $n$ chẵn thì $2 \geq 2-k=d(n-k)$ 

Do đó $d(n-k)=2$ hay $k=0$

Vậy $P(x)=x$ với mọi $x \in \mathbb{R}$




#656713 Đề thi chọn đội tuyển Quốc gia tỉnh Vũng Tàu năm 2016-2017

Gửi bởi viet nam in my heart trong 04-10-2016 - 21:54

Câu 5: b) Rõ ràng nếu gọi $A$ là tập các đoạn thẳng thỏa mãn đề bài và số phần tử của $A$ nhỏ nhất. Gọi $X$ là tập hợp tất cả các đoạn thẳng tạo từ $2$ trong số $42$ điểm trên. Khi đó $B=X \setminus A$ là một tập con của tập $X$ và nó không chứa bất cứ một tam giác nào

Do đó $|B| \leq \lfloor\dfrac{42^2}{4}\rfloor=441$ ( Theo định lý $Turan$)

Từ đó $|A| \geq \dfrac{42 \cdot 41}{2}-441=420$